The table shows the number of insects in each group. What is the combined population of the four insect groups?



Insect Population
Honeybees
7
.
3498
×
10
4

Ants
6
.
822
×
10
3

Termites
9
.
8
×
10
5

Aphids
2
.
9502
×
10
4

Answers

Answer 1

1089822 is the combined population of the four insect groups.

What is exponents?

The number of times a number has been multiplied by itself is referred to as an exponent. For instance, the expression 2 to the third (written as 2³) signifies 2 x 2 x 2 = 8. 2³ and 2 x 3 = 6 are not equivalent. Keep in mind that any number is itself when raised to the power of 1.

Given Data

Insect population

Population of honeybees = 7.3498(10⁴)

Population of honeybees = 73498

Population of ants = 6.822(10³)

Population of ants = 6822

Population of termites = 9.8(10⁵)

Population of termites = 980000

Population of aphids = 2.9502(10⁴)

Population of aphids = 29502

Combined population = 73498 + 6822+980000+29502

Combined population = 1089822

1089822 is the combined population of the four insect groups.

To learn more about exponents, visit:

https://brainly.com/question/24813773

#SPJ9


Related Questions

Giana has started collecting sea shells. Every month she adds 66​ new shells to her collection. After 66​ months of collecting, she has 3838​ shells in her collection. Using this information, create an equation written in point-slope form that models the amount of shells y​ in Giana’s collection after x​ months

Answers

She has 3838 shells in her collection after 66 months of gathering. The equation using this data is 33x+33y=1919.

Given that,

Giana has begun to gather seashells. She grows her collection by 66 new shells each month. She has 3838 shells in her collection after 66 months of gathering.

We have to create an equation using this data that models the number of shells y Giana's collection will include after x months in point-slope form.

We can write as

The total number of shells she collected =(The amount of shells she collect each month)×y+(The month of collecting the shells)×x

3838=66y+66x

66x+66y=3838

33x+33y=1919

Therefore, the equation using this data that models the number of shells Giana's collection will include after x months in point-slope form is 33x+33y=1919.

To learn more about shells visit: https://brainly.com/question/13514474

#SPJ1

Answer: give correct answers only or people on here will do the same thing u did to them bozo

Step-by-step explanation:EEEEEEEEEEEEEEEEEEEEEEEEEEEEEEEEEEEEEEEEEEEEEEEEEEEEEEEEEEEEEEEEEEEEEEEEEEEEEEEEEEEEEEEEEEEEEEEEEEEEEEEEEEEEEEEEEEEEEEEEEEEEEEEEEEEEEEEEEEEEEEEEEEEEEEEEEEEEEEEEEEEEEEEE

Algebraically determine whether the following function is Even, Odd, or Neither.
f(x)=x^3-2x
show all work please

Answers

The function is  f(x) = [tex]x^3[/tex]  + 2x is an odd function.

Given the function is :

f(x)=x^3-2x

To determine the function is Even , Odd , or Neither.

Let's Know :

What is  an Odd function?

A function f is said to be an odd function if -f(x) = f(-x), for all value of x. In Mathematics, the functions even and odd are those that satisfy specific symmetry relations, with respect to considering additive inverses.

Now, Come to the question:

According to the question;

f(x) = [tex]x^{3}[/tex] - 2x

f(-x) = (-[tex]x^3[/tex])  - 2(-x)

f(-x) = -[tex]x^3[/tex]  + 2x

Identify the relation between f(x) and f(-x)

f(x) ≠ f(-x)

Find -f(-x)

-f(-x) = -( -[tex]x^3[/tex]  + 2x)

Remove the parentheses:

-f(-x) =  [tex]x^3[/tex]  + 2x

Identify the relation:

f(x) = -f(-x)

Hence, The function is  f(x) = [tex]x^3[/tex]  + 2x is an odd function.

Learn more about Odd function at:

https://brainly.com/question/16297926

#SPJ1

How would I solve this?

Write an equation of the line containing the given point and parallel to the given line.
(8, -3); 2x - 56 = 3
Type an equation using​ slope-intercept form or using the given point in​ point-slope form.

Answers

An equation of the line containing the given point and parallel to the given line is y = 2x - 19.

The condition for two parallel lines.

In Mathematics, two (2) lines are considered to be parallel if their slopes are the same (equal) and they've different y-intercepts. This ultimately implies that, two (2) lines are parallel under the following conditions:

m₁ = m₂

Note: m is the slope.

Mathematically, the point-slope form of a line is given by this equation:

y - y₁ = m(x - x₁)

Where:

x and y are the points.m represents the slope.

Given the following equation:

2x - 56 = 3     ⇒ m₁ = 2

Therefore, m₁ = m₂ = 2.

At point (8, -3), we have:

y - y₁ = m(x - x₁)

y - (-3) = 2(x - 8)

y + 3 = 2(x - 8)

y + 3 = 2x - 16

y = 2x - 16 - 3

y = 2x - 19

Read more on slope of parallel lines here: brainly.com/question/28427398

#SPJ1

What helps the esophagus move food to the stomach?
A. Cardiac muscle contractions
B. Smooth muscle contractions
C. Skeletal and cardiac muscle contractions
D. Skeletal muscle contractions
answer asap for brainlist

Answers

[tex] \: \huge \tt{\pink{A}{\orange{N}S{\green{W}{\blue{E}{\purple{R}}}}}} : \\ \\ \: \: \rightarrow \tt \: \:B. \: Smooth \: \: muscle \: \: contractions[/tex]

Esophagus move food to the stomach with the help of Smooth muscle contractions. Thus, option B is correct.

What is esophagus?

The esophagus is the tubular structure that transports food from your mouth to your stomach. Esophagitis can cause uncomfortable swallowing and chest discomfort. Esophagitis can be caused by a number of factors. Some typical causes comprise stomach acids backing up into the esophagus, illness, oral medications, and allergies.

As food enters the stomach, the esophagus contracts. The lower esophageal sphincter (LES) is a "valve" placed directly before the stomach entrance. This valve opens to allow food to travel from the esophagus into the stomach and closes to prevent food from moving back up into the esophagus from the stomach. Hence, option B is correct.

Learn more about esophagus here:

https://brainly.com/question/22906056

#SPJ2

A car leaves Albany, NY, and travels west toward Buffalo, NY. The equation D 280 59t can be used to represent the distance, D, from Buffalo after t hours. In this equation, the 59 represents the

1. Cars distance from Albany

2. speed of the car

3. distance between Buffalo and Albany

4. numbers of hours driving

Answers

Answer:

2. Speed of the car.

Step-by-step explanation:

Given equation:

[tex]D=280-59t[/tex]

where:

D = distance from Buffalo.t = time in hours.

The car begins its journey at Albany.  

Therefore, the car is at Albany when t = 0:

[tex]t=0 \implies D=280-59(0)=280[/tex]

Therefore, 280 represents the distance between Buffalo and Albany.

After 1 hour of driving:

[tex]t=1 \implies D=280-59(1)=221[/tex]

Therefore, after 1 hour of driving, the car is 59 miles closer to Buffalo and therefore 59 miles from Albany.

After 2 hours of driving:

[tex]t=2 \implies D=280-59(2)=162[/tex]

Therefore, after 2 hours of driving, the car is 158 miles closer to Buffalo and therefore 158 miles from Albany.

Therefore, the 59 represents the speed of the car as the car travels 59 miles per hour.

Determine the number of large cups that would contain the same amount as 10 1/2 small cups if a large cup holds 1 3/4 times the quantity of liquid that's in a small cup.

Answers

The number of large cups that would contain the same amount as 10 1/2 small cups is 6.

How to illustrate the information?

From the information, it was stated that we should find the number of large cups that would contain the same amount as 10 1/2 small cups if a large cup holds 1 3/4 times the quantity of liquid that's in a small cup.

This simply means that we have to divide the values that are given. This will be illustrated as:

= 10 1/2 ÷ 1 3/4

= 10.5 / 1.75

= 6

There'll be 6 large cups.

Learn more about fractions on:

brainly.com/question/17220365

#SPJ1

#31 and 35 please show work THANK YOU!!

Answers

The parametric equations for this problem are given as follows:

31. [tex]x = t, y = \sqrt[3]{t} + 1[/tex].

35. x(t) = -1 + 3t, y(t) = 5 - 2t.

Item 31

For item 31, the parent function is the cube root function, defined as follows:

[tex]y = \sqrt[3]{x}[/tex]

From the graph of the function, it was translated two units up, hence it is defined as follows:

[tex]y = \sqrt[3]{x} + 2[/tex]

We want to parametrize the equation as a function of t, hence:

x = t.[tex]y = \sqrt[3]{t} + 2[/tex].

Item 35

A linear function is parameterized as follows:

x(t) = x(0) + at.y(t) = y(0) + at.

The initial point of the function, at t = 0, is (-1, 5), hence:

x(0) = -1, y(0) = 5.

After 1 second, they moved to point (2,3), hence the coefficients are given as follows:

a = 2 - (-1) = 3.b = 3 - 5 = -2.

Thus the parameterized equations are given as follows:

x(t) = -1 + 3t.y(t) = 5 - 2t.

More can be learned about parametric equations at https://brainly.com/question/27247899

#SPJ1

Giving a test to a group of students, the grades and gender are summarized below A B C TotalMale 5 8 14 27Female 13 20 10 43Total18 28 24 70If one student was chosen at random, find the probability that the student got an A or a B on the test. Write your answer as a reduced fraction or whole number.P(A or B)=

Answers

Anime Girl, this is the solution:

We have a total of 70 students (male and female), in consequence:

• Students that got an A = 18

,

• Students that got a B = 28

,

• Stiudents that got a C = 24

• P(A or B) = A + B / Total students

• P(A or B) = 18 + 28/70

,

• P(A or B )= 46/70

,

• P(A or B) = 23/35 (Simplifying) or 0.66

Help please asap asap

Answers

For this problem, we are given the dimensions of a right triangle, we need to apply the trigonometric relation to determine which one results in 3/5.

We have three possible options sin, cos, and tan. The sine is the division between the opposite cathetus and the hypothenuse, which for this case is equal to 3/5, therefore it is the correct option.

The answer is A. sin.

3. For the number line shown, which
statement is NOT true?

Answers

Answer: A.

Step-by-step explanation:

D is not less than c because c is a negative number.

(x^3y^5)^3 × -x^6y^5

Answers

To simplify the expression you can first use this property of exponents

[tex](a\cdot b)^m=a^mb^m[/tex]

Then you have

[tex]\begin{gathered} (x^3y^5)^3=(x^3)^3(y^5)^3 \\ (x^3y^5)^3=x^{3\cdot3}y^{5\cdot3} \\ (x^3y^5)^3=x^9y^{15} \end{gathered}[/tex]

Now apply this property of exponents

[tex]a^b\cdot a^c=a^{b+c}[/tex]

Therefore, you have

[tex]\begin{gathered} (x^3y^5)^3\cdot-x^6y^5=x^9y^{15}\cdot-x^6y^5 \\ (x^3y^5)^3\cdot-x^6y^5=x^9\cdot-x^6\cdot y^{15}\cdot y^5 \\ (x^3y^5)^3\cdot-x^6y^5=-x^{9+6}\cdot y^{15+5} \\ (x^3y^5)^3\cdot-x^6y^5=-x^{15}\cdot y^{20} \\ (x^3y^5)^3\cdot-x^6y^5=-x^{15}y^{20} \end{gathered}[/tex]

Pls help i need this ASAP!

Answers

The solution for the given inequalities are as follows

(17)k≥1     ,(18)c > 14      ,(20)y ≥ 2      ,(21)z < 9     ,(23)x < 1    ,(24)v ≤ -1  .

In the question ,

it is given that

(17) the inequality

3(k-5)+9k≥-3

3k-15+9k ≥ -3

12k -15 ≥ -3

12k ≥ 15-3

12k ≥ 12

k≥1

(18) the inequality

-(7c-18)-2c > 0

-7c+126-2c>0

-9c+126>0

-9c > -126

c > 126/9

c > 14

(20)given the inequality

-4 ≤ 4(6y-12)-2y

-4 ≤ 24y - 48 -2y

-4 ≤ 22y - 48

48-4 ≤ 22y

44/22 ≤ y

y ≥ 2

(21) given the inequality

30 > -(5z+15)+10z

30 > -5z -15 +10z

30 > 5z-15

45 > 5z

45/5 > z

z < 9

(23) given the inequality

4x+3 < 3x+6

4x-3x < 6-3

3x < 3

x < 1

(24) given the inequality

4v+8 ≥ 6v+10

-10+8 ≥ 6v-4v

-2 ≥ 2v

-2/2 ≥ v

-1 ≥ v

v ≤ -1

Therefore , the solution for the given inequalities are (17)k≥1     ,(18)c > 14      ,(20)y ≥ 2      ,(21)z < 9     ,(23)x < 1    ,(24)v ≤ -1  .

Learn more about Inequality here

https://brainly.com/question/17135872

#SPJ1

When you sold your shares, your account showed $4680.00 ($72.00 × 65 shares), for a total gain of $357.50 ($4680.00 – $4322.50). What percentage gain did you experience?

Answers

The percentage that 7.64% gain experienced you if sold your shares, your account showed $4680 for a total gain of $357.50.

What is the percentage?

The percentage is defined as a ratio expressed as a fraction of 100.

For example, If Saima obtained a score of 57% on her exam, that corresponds to 67 out of 100. It is expressed as 57/100 in fractional form and as 57:100 in ratio form.

Given that when you sold your shares, your account showed $4680.00 ($72.00 × 65 shares)

For a total gain of $357.50 ($4680.00 – $4322.50).

Let the percentage gain you experienced be x %

As per the given data, the solution would be as:

⇒ x% of 357.50 = 4680.00

x%  is expressed as x/100 in fractional form

⇒ (x/100)(357.50) = 4680.00

⇒ x = 357.50/4680.00 × 100

⇒ x = 7.64%

Therefore, the percentage that 7.64% gain experienced you.

Learn more about the percentages here:

brainly.com/question/24159063

#SPJ1

The coldest recorded temperature in the United States is

80°F, in Alaska. The warmest recorded temperature in the United States is 134°F, in California.
How much higher is the warmest recorded temperature than the coldest recorded temperature?

Answers

The warmest recorded temperature is 214°F higher than the coldest temperature.

Since the sign of the two numbers are same, their final values will be added and the result will have the same signs as the two numbers (or the one with the greater absolute value). This is called the multiplicative property.

Expression for difference between the two temperatures is,

= 134−(−80)

By applying the multiplicative property of negative sign, we get

=134+80

= 214°F

Therefore, we conclude that the warmest recorded temperature is 214°F  higher than the coldest temperature.

To learn more about multiplicative property

visit; https://brainly.com/question/386524

#SPJ1

Before art class starts, Gordon makes sure that each table has a full bowl of paint. He has 1/2 of a gallon of paint and is able to fill the bowls for all 7 tables. How many gallons of paint does each table have?

Answers

Answer:0.07142857142

Step-by-step explanation:

0.07142857142 because 1/2/7= or .5/7=0.07142857142

.5= how much paint

7= how many tables that are needed to be filled

0.07142857142= how much paint

each table will receive

HELPP
4,250/204 = y/528

Answers

Hello! So....

We are given the following:

[tex]\frac{4250}{204}= \frac{y}{528}[/tex]

_______________________

1. Switch sides.

[tex]\frac{y}{528} =\frac{4250}{204}[/tex]

_______________________

2. Cancel the common factor (34).

[tex]\frac{y}{528}= \frac{125}{6}[/tex]

_______________________

3. Multiply both sides by 528.

[tex]\frac{528y}{528} =\frac{125*528}{6}[/tex]

_______________________

4. Simplify.

[tex]y=11,000[/tex]

^Hence, our solution.

____________________________________

Hope this helps! If so, please lmk! Thanks and good luck!

4,250/204 = y/528

Multiply both sides of the equation by 8976, the lowest common denominator of 204,528.

44×4250 = 17y

Multiply 44 and 4,250 to get 187,000.

187000 = 17y

Swap the sides so that all the terms of the variables are on the left side.

17y = 187000

Divide both sides by 17.

y = 187000/17

Divide 187,000 by 17 to get 11,000.

y=11000

The purchase price of a home is $150,000 what are the total closing cost if the following fees apply 5% purchase price title search $300 the buyer pays only half the fee low cost 4% recording $65 in taxes 1,235

Answers

It should be noted that the purchase price of the home will be $160865.

How to illustrate the information?

Based on the information, the following can be illustrated:

Purchasing price = $150000

Purchase price fee = 5% × $150000 = $7500

Title search = $300

Love cost = 1/2 × 4% × $150000 = $3000

Taxes = $65

Total amount = $150000 + $7500 + $300 + $3000 + $65

= $160865

Learn more about tax on:

brainly.com/question/25783927

#SPJ1

i have a derivative question about tangent lines, pic included

Answers

To find the horizontal tangent line, we have to conduct the first derivative test. The first derivative test sets the derivative of the function equal to 0.

Let's start out with finding the derivative of the function:

It seems like we will have to use Quotient Rule.

[tex]f^{\prime}(x)=\frac{(x^2-7x+1)(x+9)^{\prime}+(x^2-7x+1)^{\prime}(x+9)}{(x^2-7x+1)^2}[/tex]

Now we can use basic derivative rules and properties to find the final derivative:

[tex]f^{\prime}(x)=\frac{(x^2-7x+1)+(2x-7)(x+9)}{(x^2-7x+1)^2}[/tex]

Setting the derivative equal to 0, we can begin the first derivative test:

[tex]0=\frac{(x^{2}-7x+1)+(2x-7)(x+9)}{(x^{2}-7x+1)^{2}}[/tex]

We can simplify the derivative before solving (expand and combine like terms):

[tex]0=\frac{-(x^2+18x-64)}{(x^2-7x+1)^2}[/tex]

From here, we need to determine our values of x that will make the derivative equal to 0 or undefined. In this case, since we are not trying to find relative maxima or minima, we just need to find where the entire derivative is equal to 0. We can simply just graph the numerator and find its roots:

We can see that the values of x will be -21.0416 and 3.04159 will be the values that make the derivative equal to 0.

Also recall what the derivative is. The derivative is the slope of the tangent line. In order to find the horizontal tangent line, we know that horizontal tangent lines' slope is equal to 0.

Therefore, we can use either value of x to now find our equation.

Recall what the point-slope form of a line is:

[tex]y-y_1=m(x-x_1)[/tex]

Since we know that the derivative slope must equal 0, then we know that m = 0. We now need to find a point (x, y). Substitute one of the x values into the original equation to find an actual point on the function (remember not to round intermediate values on your calculator!):

[tex]\begin{gathered} f(3.04159)=\frac{3.04159+9}{3.04159^2-7(3.04159)+1} \\ f(3.04159)=-1.09074 \end{gathered}[/tex]

Now we have a point (x, y) (I will put it in the actual radical form here):

[tex]\bigg(\sqrt{145}-9,\frac{-2\sqrt{145}}{45}-\frac{5}{9}\bigg)[/tex]

Substitute our variables into point-slope form:

[tex]y-\bigg(\frac{-2\sqrt{145}}{45}-\frac{5}{9}\bigg)=0\bigg(x-(\sqrt{145}-9)\bigg)[/tex]

Simplify and rewrite to obtain our final answer:

[tex]y=\frac{2\sqrt{145}}{45}+\frac{5}{9}[/tex]

what is an equation

Answers

We have to write the equation of the line that pass through (4,-5) and (2,0).

We can calculate the slope as:

[tex]m=\frac{y_2-y_1}{x_2-x_1}=\frac{0-(-5)}{2-4}=\frac{5}{-2}=-2.5[/tex]

We can then write the slope-point form of the equation as:

[tex]\begin{gathered} y-y_0=m(x-x_0) \\ y-0=-2.5(x-2) \\ y=-2.5x+5 \end{gathered}[/tex]

Answer: y=-2.5x+5

Roxy created a table listing some of her fixed expenses for the first three months of the year. Expense Jan Feb Mar emergency fund $50 $50 $50 cell phone $89 $89 $89 internet $55 $55 $55 rent payment $986 $986 $986 transportation $30 $30 $30 insurance $15 $15 $15 gym membership $24 $24 $24 entertainment $35 $35 $35 Which pie chart accurately represents the data in the table? PLATO

Answers

The amount in Roxy's table listing of expenses gives the ratio of the items in the pie chart as follows;

Emergency fund; 3.89%

Cell phone; 6.93%

Internet; 4.28%

Rent payment; 76.79%

Transportation; 2.34%

Insurance; 1.17%

Gym membership; 1.97%

Entertainment; 2.73%

The pie chart can then be constructed using those percentage as a proportion of 100

Please see the attached pie chart of the expenses created with Sheets

What is a pie chart?

A pie chart illustrates the numerical proportion of items in a dataset by using a circular graphic that is divided into sector slices

The given values in the question are;

Emergency fund = $50

Cell phone = $89

Internet = $55

Rent payment = $986

Transportation = $30

Insurance = $15

Gym membership = $24

Entertainment = $35

The total cost of the fixed expense is therefore;

$50+$89+$55+$986+$30+$15+$24+$35 = $1284

The proportion of each component in the pie chart is found as follows;

[tex] \displaystyle{Emergency \: fund = \frac{50}{1284} \times 100 \approx 3.89 \%}[/tex]

[tex] \displaystyle{Cell \: phone = \frac{89}{1284} \times 100 \approx 6.93 \%}[/tex]

[tex] \displaystyle{Internet = \frac{89}{1284} \times 100 \approx 4.28 \%}[/tex]

[tex] \displaystyle{Rent \: payment = \frac{986}{1284} \times 100 \approx 76.79 \%}[/tex]

[tex] \displaystyle{Transportation = \frac{30}{1284} \times 100 \approx 2.34 \%}[/tex]

[tex] \displaystyle{Insurance = \frac{15}{1284} \times 100 \approx 1.17 \%}[/tex]

[tex] \displaystyle{Gym \: membership = \frac{24}{1284} \times 100 \approx 1.87 \%}[/tex]

[tex] \displaystyle{Entertainment = \frac{35}{1284} \times 100 \approx 2.73 \%}[/tex]

Using the above percentages, the pie chart can be constructed

Learn more about data presentation as a pie chart here:

https://brainly.com/question/11154495

#SPJ1

Rick's car gets 29.7 miles per gallon on the highway. If his fuel tank holds 10.45 gallons, then how far can he travel on one full tank of gas?

Answers

We will investigate how to use the mileage meter of a car to determine the amount of distance a car can travel per given fuel.

Mileage is a economic performance parameter of a car that gives the fuel economy. It is usually expressed as a ratio of:

[tex]\text{Mileage = }\frac{Total\text{ Distance Travelled}}{Total\text{ Fuel consumed}}[/tex]

The mileage of a car is given as follows:

[tex]\text{Mileage = 29.7 }\frac{miles}{gallon}[/tex]

We can say that this car can travel 29.7 miles per gallon on highway.

We now stop at the petrol station for re-fueling. We fuel up the car to:

[tex]\text{Fuel = 10.45 gallons}[/tex]

We need to determine the distance that this car can travel for the amount of re-fueling. This is where we can use the mileage to determine the distance the car can travel as follows:

[tex]\begin{gathered} \text{Distance = Mileage}\cdot Fuel \\ \end{gathered}[/tex]

Plug in the respective quantitites and solve for the distance that the car can travel as follows:

[tex]\begin{gathered} \text{Distance = 29.7}\frac{miles}{gallon}\cdot10.45\text{ gallons} \\ \\ \textcolor{#FF7968}{Dis\tan ce}\text{\textcolor{#FF7968}{ = 310.365 miles}} \end{gathered}[/tex]

For the amount of re-fueling the distance travelled is 310.365 miles

Correct answer answer now please for brainlist asap

Answers

Answer:

D. Organ systems

Step-by-step explanation:

Homeostasis is maintained by the nervous and endocrine systems.

Need help for the questions need help with b and c

Answers

b) No, it is not

c) x + 1 is a factor

Explanation:

[tex]b)\text{ }f(x)=x^3-7x^2+8x\text{ + 16}[/tex]

To determine if 10 is a function, we substitute x with 10 in the function:

[tex]\begin{gathered} f(10)\text{ = 1}0^3-7(10)^2\text{ + 8(10) + 16} \\ f(10)\text{ = 1000-700+80+16} \\ f(10)\text{ = }396 \end{gathered}[/tex]

No, it isn't a . This is because f(10) is not equal to zero, it is not a zero o. It has a remanider of 396.

c) Using synthetic division:

for x + 1: to be a factor, x = -1

+1 1 0

The last number is equal to zero. This indicates the remainder is zero.

Hence, x + 1 is a factor of the function f(x)

Mary multiplied a number by-3.When she added the result to 6,she got 12. What is her number?

Answers

Explanation

Step 1

set the equation

a) let x represents the number

[tex]number=x[/tex]

b) now , translate

[tex]i)Mary\text{ multiplied a number by 3}\Rightarrow x*3=3x[/tex]

then

[tex]\begin{gathered} ii)shed\text{ added the result to 6} \\ 3x+6 \end{gathered}[/tex]

finally

[tex]\begin{gathered} iii)\text{ she got 12, hence} \\ 3x+6=12 \end{gathered}[/tex]

therefore, the equation is

3x+6=12

Step 2

now, solve the equation

[tex]3x+6=12[/tex]

The subtraction property of equality states that if the same number is subtracted from both sides of an equation, then the equality still holds, so we can subtract 6 in both sides in order to isolate the term where x is, hence

[tex]\begin{gathered} 3x+6=12 \\ 3x+6-6=12-6 \\ 3x=6 \end{gathered}[/tex]

finally, apply the division property of equality and divide both sides by 3

[tex]\begin{gathered} 3x=6 \\ \frac{3x}{3}=\frac{6}{3} \\ x=2 \end{gathered}[/tex]

so, the number is 2

I hope this helps you

10. An express train is travelling at 80 km/h. How far does it go in:
(a) 1 minute (b) 1 second?​

Answers

There are 60 minutes per hour. So divide by 60 to find km/minute.

80 km / h*60m/h = 80/60 km/m = 1.33 km/m.

Under a rotation about the origin, the point A ( 5 , − 1 ) is mapped to the point A ' ( 1 , 5 ) . What is the image of the point B ( − 4 , 6 ) under this rotation? Explain.

Answers

The image of the point B ( − 4 , 6 ) under 270° clockwise rotation is point (-6 , -4).

A transformation that revolves a figure around a point is called a rotation. The rotational center is where we refer to it. The shape and dimensions of a figure remain the same while facing in a different direction. You can rotate a figure either clockwise or counterclockwise.

Under a rotation about the origin, the point A ( 5 , − 1 ) is mapped to the point A ' ( 1 , 5 ) is a -

270° clockwise rotation: (x,y) becomes (-y,x)

The image of the point B ( − 4 , 6 ) under this rotation is

(x , y) = (-y , x)

i.e.  (-6 , -4)

Assuming that the rotation's center is at (0, 0). Here are the guidelines for rotation:

90° clockwise rotation: (x,y) becomes (y,-x)90° counterclockwise rotation: (x,y) becomes (-y,x)180° clockwise and counterclockwise rotation: (x, y) becomes (-x,-y)270° clockwise rotation: (x,y) becomes (-y,x)270° counterclockwise rotation: (x,y) becomes (y,-x)

Therefore , The image of the point B ( 4, 6) when rotated 270 degrees clockwise is point (-6 , -4).

To learn more about Rotation

https://brainly.com/question/16691874

#SPJ1

Aisha paid $33.15 for the pizza for her and her two friends. How much did it cost each of them for the pizza?

Answers

Answer:$11.05

Step-by-step explanation:

Answer:

$11.05 each

Step-by-step explanation:

To find x, the cost between Aisha and her two friends you must divide the total amount by the number of people.

There are three people, her and two friends and the total cost for the pizza is $33.15

$33.15 ÷ 3 = $11.05

dses 12.4. lete the following: Find the intercepts and domain and perform the symmetry test on each parabola with equations. (n) = 8 (d) x - 4y the vertex, focus, and endnoin

Answers

Find the intercepts and domains

1. Intercepts

x-intercept (when y = 0)

x = 0

y-intercept (when x = 0)

y = 0

Domain

y = All real numbers

Symmetry

Yes it is symmetry

Tyler was making a meal for his family. For every 3/4 pound of beef, the recipe calls for 9/8 cups of broccoli. How many cups of broccoli would be needed for a pound of beef?

a1/2 cup of broccoli per beef

b2/3 pound of broccoli per beef

c1 and ½ cup of broccoli per beef

d5/4 cup of broccoli per beef

Answers

The number of cups of broccoli that would be needed for a pound of beef is C. 1 1/2 cup of broccoli per beef.

How to calculate the value?

From the information, it was stated that Tyler was making a meal for his family and that for every 3/4 pound of beef, the recipe calls for 9/8 cups of broccoli.

Therefore, the cups of broccoli that would be needed for a pound of beef will be:

3/4/1 = 9/8/x.

where x = cups of broccoli

Cross multiply

3/4x = 9/8

x = 9/8 ÷ 3/4

x = 9/8 × 4/3

x = 3/2

x = 1 1/2cups

Therefore, the correct option is C.

Learn more about fractions on:

brainly.com/question/17220365

#SPJ1

Answer:

[tex]\textsf{c) \quad $1 \frac{1}{2}$ cups of broccoli per 1 lb of beef}.[/tex]

Step-by-step explanation:

Given information:

For every ³/₄ lb of beef, the recipe calls for ⁹/₈ cups of broccoli.

Create a ratio of pounds of beef to cups of broccoli, where x is the number of cups of broccoli to one pound of beef:

[tex]\implies \dfrac{3}{4}: \dfrac{9}{8}=1:x[/tex]

[tex]\implies \dfrac{\frac{3}{4}}{\frac{9}{8}}=\dfrac{1}{x}[/tex]

Cross multiply:

[tex]\implies \dfrac{3}{4}x=\dfrac{9}{8}[/tex]

[tex]\implies x=\dfrac{9}{8} \div \dfrac{3}{4}[/tex]

To divide fractions, flip the second fraction (make the numerator the denominator, and the denominator the numerator) then multiply it by the first fraction:

[tex]\implies x=\dfrac{9}{8} \times \dfrac{4}{3}[/tex]

[tex]\implies x=\dfrac{9 \times 4}{8 \times 3}[/tex]

[tex]\implies x=\dfrac{36}{24}[/tex]

[tex]\implies x=\dfrac{3 \cdot \diagup\!\!\!\!\!\!12}{2 \cdot \diagup\!\!\!\!\!\!12}[/tex]

[tex]\implies x=\dfrac{3}{2}[/tex]

[tex]\implies x=1 \frac{1}{2}[/tex]

Therefore, 1¹/₂ cups of broccoli would be needed to one pound of beef.

Solve the following question show your work

Answers

Solution:

iven :

[tex]7\frac{1}{5}\times4\frac{5}{6}[/tex]

To solve,

step 1: Convert the mixed fractions into improper fractions.

[tex]\begin{gathered} Given\text{ an improper fraction of the form: a}\frac{b}{c},\text{ to convert to an } \\ improper\text{ fraction, we have} \\ \frac{(c\times a)+b}{c} \\ \end{gathered}[/tex]

Thus,

[tex]\begin{gathered} 7\frac{1}{5}=\frac{(7\times5)+1}{5}=\frac{35+1}{5}=\frac{36}{5} \\ 4\frac{5}{6}=\frac{(4\times6)+5}{6}=\frac{24+5}{6}=\frac{29}{6} \end{gathered}[/tex]

step 2: Perform the operation of multiplication between both fractions.

Thus,

[tex]\begin{gathered} \frac{36}{5}\times\frac{29}{6}=\frac{36\times29}{5\times6} \\ cancel\text{ out common factor,} \\ =\frac{6\times29}{5} \\ =\frac{174}{5} \end{gathered}[/tex]

step 3: Convert the resulting improper fraction into a mixed fraction.

Thus,

[tex]\frac{174}{5}=34+\frac{4}{5}[/tex]

Hence, the solution is

[tex]34\frac{4}{5}[/tex]

Other Questions
Virtual Lab: Relative and Absolute Dating use the ruler to determine the quotients of 1 divided 1/10 and 4 divided 1/10 What was the result of the falling out among Greek city-states after the Persian War? Calculate the centripetal force exerted on a 900 kg car that negotiates a 500 m radius curve at 25.0 m/s. B) Find the minimum static friction coefficient between the tires and the road. What is the ATOMIC NUMBER for an element with 5 protons, 6 neutrons and 2 electrons? Which of the following can be considered mechanical energy? A. a hot bowl of soup B. plant growth C. doing bicep curls Your body is covered with skin Its main fucantion is to protect the organs inside the body. It also function for insulation regulation of heat, and excretion of waster. Make a word that means related to winter spring summer and fall Which of these is a prepositional phrase?the distant futurereceive the awarddribble the ballunder the window Which of the following increases the motion of atoms and molecules? Which of the following atoms has the smallest radius?The Periodic TableA. Cesium (Cs)B. Potassium (K)C. Sodium (Na)D. Lithium (Li) CThe lack of shelter and inadequate resources proved a major test for the newContinental Army.Many soldiers die of diseases or desert.Washington used the time to train and discipline his troops.The list above is most closely associated with the -A Battle of Lexington & ConcordB Battle of SaratogaC Winter at Valley ForgeD Battle of Yorktown Each circled letter inthe circuit diagram represents a meter that is used tomeasure a quantity in the circuit. Which meters show the amount of chargepassing a point each second?A. A and CB. A and BC. B and DD. C and D In the new code of laws which I suppose it will be necessary for you to make, I desire you would remember the ladies and be more generous and favorable to them than your ancestors. Do not put such unlimited power into the hands of the husbands. Remember, all men would be tyrants if they could. Abigail Adams, 1776 What is the authors main argument in this passage? Social positions should continue to be inherited. Greater rights for women should be secured. The system of primogeniture should be abandoned. The practice of arranged marriages should be rejected. 3.- The north and south poles of asolenoid change withA the type of material in the coreB the number of loops in the coilC the tightness of loops in the coilD the direction of the current Which system of inequalities is graphed below? Enulose 10g/15mL Sig: 2 tbsp po bid x 3d - how many ml is the patient taking per day? Purple hibiscusChapter 6Page. 78: Papa invites Ifeoma and her three children to spend the Christmas day with them. Summarize the short conversation that follows. You have multiple manufacturing computers that control the machinery to several assembly lines. The software for the assembly line controls rarely changes. The assembly lines cannot go down because of problem Windows updates or new features.Q. What do you recommend? How do the respiratory and circulatory systems change throughout the day? I will give brainiest to best answer!